Der Abstand zwischen Potenz von 3 und der größten Potenz von 2

Für eine positive ganze Zahl k 1 , der größte Bitindex von 3 k wird von gegeben

M k Protokoll 2 3

Der Abstand zwischen 3 k Und 2 M kann geschrieben werden als

3 k 2 M A k 2 M
Wo 0 < A k < 1 .

Meine Fragen sind:

  1. Ist es wahr dass A k 10 C , Wo C ist eine positive Konstante. Wie kann ich das beweisen oder widerlegen?
  2. Wenn Punkt 1 wahr ist, wie schätze ich die Konstante? C ?

Als Randnotiz habe ich numerisch nachgerechnet k bis 10.000. Es scheint, dass der Wert von A k in einem stabilen Bereich gehalten ( 0 , 1 ) , und es könnte so klein sein wie Ö ( 10 6 ) .

Ich denke, das ( mathoverflow.net/questions/116840/… ) beweist, dass es so etwas nicht gibt C
@ Arthur Es gibt wirklich keinen offensichtlichen Grund. C könnte definitiv nicht ganzzahlig sein, glaube ich. Ich werde meine Frage bearbeiten und aktualisieren, um dieses Limit zu entfernen.
Die kleinsten Werte von A wird wann sein M / k ist ein ungerader fortgesetzter Bruch von Protokoll 2 3.
@Exodd Danke für den Link. Ja, es beweist, dass die Untergrenze von A kommt drauf an M . Die Schätzung der Konstante in diesem Link stimmt jedoch nicht genau mit dem überein, was ich numerisch beobachtet habe, wenn ich nichts übersehen habe.
Seit A kommt drauf an k . Sie sollten es als schreiben A k
Sie können diese Frage zu rationalen Darstellungen von genießen Protokoll 2 ( 3 ) math.stackexchange.com/q/3981318/207316
@ThomasAndrews Ich habe es zum Aktualisieren erneut bearbeitet A -> A k .

Antworten (2)

Wenn M k ist ein ungerader fortgesetzter Bruch für Protokoll 2 3 Dann:

0 Protokoll 2 3 M k < 1 k 2

Dann

3 k = 2 k Protokoll 2 3 < 2 M + 1 k

So:

3 k 2 M < 2 M + 1 / k 2 M = 2 M ( 2 1 / k 1 )

So: A = A k 2 1 / k 1. Das können wir beliebig klein machen.

Die fortgesetzten Brüche sind unendlich, und die Hälfte davon wird ungerade sein. (Die Terme für gerade Kettenbrüche geben Ihnen Beispiele, wo 2 M > 3 k aber der Unterschied ist gering.)

Wann zum Beispiel k = 12 , M = 19 , A k .0136 . Wenn k = 53 , M = 84 , Dann A k 0,00209.


Wenn M / k ist ein gerader fortgesetzter Bruch für Protokoll 2 3 , Dann

2 M > 3 k > 2 M 1
So:

3 k = 2 k Protokoll 2 3 > 2 M 1 k
Und:
3 k 2 M 1 > 2 M 1 ( 2 1 1 / k 1 )

So für M / k ein gerader fortgesetzter Bruch, wie 8 / 5 oder 65 / 41 , du erhältst A k 1.

Es gibt also Werte von A k willkürlich nahe 0 , Und A k willkürlich nahe 1.

Ich folge nicht der ersten Ungleichung 0 Protokoll 2 3 M k < 1 k 2 . Mir ist die ungerade Kettenbrucherweiterung für nicht bekannt l Ö G 2 3 , entweder. Als ich Ihre Werte numerisch überprüft habe. Es scheint, wann k l Ö G 2 3 hat den Bruch kleiner als 0,5 , das Erhaltene M / k nennt man den ungeraden Kettenbruch l Ö G 2 3 . Könnten Sie bitte ein wenig erklären, wie Sie die erste Ungleichung erhalten können?
Die Kettenbrüche für eine irrationale Zahl a sind eine Folge rationaler Zahlen P N / Q N mit | a P N . / Q N | < 1 / Q N 2 . Wenn N ist ungerade, P N / Q N < a , Wenn N ist dann eben P N / Q N > a . Es gibt viel zu lesen über Kettenbrüche - diese sind nicht typisch M , k aber nur eine spezielle Gruppe von Fällen, in denen wir "gute" Annäherungen erhalten Protokoll 2 3. @J.Doe
In diesen Fällen, k Protokoll 2 3 wird "nah" an einer ganzen Zahl sein und nicht nur Bruchteile haben < 0,5. Der Punkt ist, wir können immer größer finden k mit dem Bruchteil von k Protokoll 2 3 weniger als 1 k .
Ich habe noch eine Frage. Im Endergebnis ergibt sich der ungerade Kettenbruch für l Ö G 2 3 , nämlich A = A k 2 1 / k 1. Hier wissen wir, dass es keine Untergrenze für gibt A k . Gibt es jedoch eine Möglichkeit zu zeigen, dass die 2 1 / k 1 , oder so ähnlich, gibt an, wie schnell der kleinste Wert von ist A k 0 ? Ich weiß Ihre Antwort wirklich zu schätzen.
Bei der letzten Frage gibt es nicht wirklich einen Weg. Es ist möglich dass Protokoll 2 3 M / k < 1 / k 3 oder noch kleiner für unendliche Werte von k . @J.Doe
Bitte ertragen Sie meine Unwissenheit zu diesem Thema. Ist die Existenz unendlicher Werte von k Das " l Ö G 2 3 M / k < 1 / k 3 oder noch kleiner" irgendwo ein bewiesenes Ergebnis?
Ich sagte: „Es ist möglich.“ Es ist mit ziemlicher Sicherheit unbekannt.

Es gibt kein N so dass A k ist unterhalb 1 / 3 für alle k > N .

Zum Beispiel A k < ( 1 / 3 ) für einige k . Dann 2 M < 3 k < ( 4 / 3 ) 2 M . Mal 3 zu bekommen ( 3 / 2 ) 2 M + 1 < 3 k + 1 < 2 M + 2 Erzwingen des nachfolgenden Werts, A k + 1 , überschreitet 1 / 2 .

Hmm, interessant. Aber hier ist ein Zahlenbeispiel, das zu widersprechen scheint. k = 31867 , M = 50508 , A k = 0,00000726 .
@J.Doe Diese Antwort ist ungefähr alles k . Es kann einzelne kleine Beispiele geben und dies würde immer noch zutreffen.
Oscar, ich hoffe, meine Bearbeitung ist in Ordnung.
Verwenden Sie einfach den Euklid-Algorithmus, um Vielfache von log_2 (3) zu finden, die willkürlich nahe an einer Ganzzahl liegen.